Démonstration (dérivabilité)

Bonjour,

Cette démonstration vous semble t-elle correcte ?

Théorème : Théorème de bijection et condition de dérivabilité en b = f(a) de la fonction réciproque

Soit f une fonction continue, strictement monotone de I vers J. Soit a $\in$ I tel que f'(a) $\neq$ 0. Si $f^{-1}$ est dérivable en $b = f(a)$ on a, $(f^{-1})'(b)=\frac{1}{f'(a)} = \frac{1}{f'(f^{-1}(b))}$ car en effet $a = f^{-1}(b)$

Démonstration :

Montrons que : $lim_{x\longrightarrow b}\frac{(f(x)-f(b)}{x-b} = f'(b)$ (condition de dérivabilité)

$\frac{f^{-1}(x) - f^{-1}(b)}{x-b} = \Bigg(\frac{f (f^{-1}(x)) - f (f^{-1}(b))}{f^{-1}(x) - f^{-1}(b)}\Bigg)^{-1}$

or $lim_{x\longrightarrow b} f^{-1}(x) = f^{-1}(b) = a$

On en déduit :

$lim_{x\longrightarrow b} \Bigg(\frac{f (f^{-1}(x)) - f (f^{-1}(b))}{f^{-1}(x) - f^{-1}(b)}\Bigg)^{-1} = (f'(b))^{-1}$

Merci d'avance.

Réponses

  • Ça commence mal, on suppose seulement $f$ continue et monotone, et on parle de $f'(a)$... La première ligne de la démonstration est un peu malheureuse également, ce n'est pas ce que tu cherches à montrer ! À la fin tu trouves $\frac{1}{f'(a)}$ et pas $\frac{1}{f'(b)}$, qui n'a pas de sens en général.
  • La première ligne de la démonstration, c'est moi qui l'ai rajouté, parce que je pensais que c'était ce que le prof cherchait à démontrer. En fait, je pense que je n'ai pas compris le résonnement du prof.
    En suivant tes conseils ...

    Théorème : Théorème de bijection et condition de dérivabilité en b = f(a) de la fonction réciproque

    Soit f une fonction continue, strictement monotone de I vers J. Soit $b \in I$ tel que $ f'(b) \neq 0$. etc

    Démonstration :

    $\frac{f^{-1}(x) - f^{-1}(b)}{x-b} = \Bigg(\frac{f (f^{-1}(x)) - f (f^{-1}(b))}{f^{-1}(x) - f^{-1}(b)}\Bigg)^{-1}$
    $lim_{x\longrightarrow b} \Bigg(\frac{f (f^{-1}(x)) - f (f^{-1}(b))}{f^{-1}(x) - f^{-1}(b)}\Bigg)^{-1} = \frac{1}{f'(a)} = (f^{-1})'(b)$ (ça c'est bien la définition du nombre dérivé de f-1 en b ?)

    Donc, $f^{-1}$ est dérivable en b (c'est ce que l'on cherche à démontrer en fait)

    Peut tu m'éclairer le début de la démonstration ?
  • Sous réserve d'existence, on a : $$(f^{-1})'(b) =
    \lim_{x \to b}
    \frac{f^{-1}(x) - f^{-1}(b)}{x-b}
    $$ On passe à l'inverse pour réécrire : $$
    \frac{x-b}{f^{-1}(x) - f^{-1}(b)}
    = \frac{f(f^{-1}(x)) - f (f^{-1}(b))}{f^{-1}(x) - f^{-1}(b)}
    $$ On reconnaît le taux d'accroissement de $f$ entre les points $y=f^{-1}(x)$ et $a = f^{-1}(b)$.
    Il vient (détails techniques) $$
    \lim_{x \to b} \frac{x-b}{f^{-1}(x) - f^{-1}(b)} =
    \lim_{y \to a} \frac{f(y)-f(a)}{y-a} = f'(a).
    $$ Ainsi $$
    (f^{-1})'(b) =
    \lim_{x \to b}
    \frac{f^{-1}(x) - f^{-1}(b)}{x-b} = \frac{1}{f'(a)}$$
  • @marsup @Poirot
    C'est tout de suite infiniment plus clair, merci beaucoup ;-)
  • Sinon, tu dérives la composée $f^{-1}\circ f$.
Connectez-vous ou Inscrivez-vous pour répondre.